Download Slide 1

Document related concepts

Infection wikipedia , lookup

Focal infection theory wikipedia , lookup

Dental emergency wikipedia , lookup

Adherence (medicine) wikipedia , lookup

Dysprosody wikipedia , lookup

Infection control wikipedia , lookup

List of medical mnemonics wikipedia , lookup

Transcript
What is the most common site of origin of ocular
sebaceous carcinoma?
(A) Meibomian gland
(B) gland of Zeis
(C) lacrimal gland
(D) caruncle
(E) multicentric origin
Answer A.
Explanation: Sebaceous carcinoma, also called
Meibomian gland carcinoma, represents an
aggressive primary malignancy of the adnexal
epithelium of sebaceous glands.
The tumor most frequently develops on the upper
eyelid of elderly patients.
Sebaceous carcinoma carries a poor prognosis
because of its high recurrence rate and tendency to
metastasize. At presentation, 25% of patients will
already have regional lymph node involvement.
Early diagnosis and subsequent surgical therapy
may lead to higher survival rates. Standard surgical
excision or Mohs surgery may be used as primary
therapy. Radiation therapy is reserved for
nonsurgical patients.
Depicted in Fig. 38-2 is an 18-year-old male whose
left eye was struck with a baseball. He complains of
double vision, a headache, and tenderness in his left
cheek. The most likely injury is
(A) orbital floor fracture
(B) intracranial hemorrhage
(C) retinal detachment
(D) LeFort III fracture
Answer A.
Explanation: This most likely represents an
orbital floor fracture. A black eye finding on
physical examination is not an innocuous finding.
It must be followed through with a thoroughly
directed head/neck examination including
cranial nerves II through XII, palpation for
tenderness, and an ophthalmologic consultation.
A CT scan with coronal and axial scanning cuts is
recommended
Typically after localized trauma to the periorbital
region, bony sheer forces create an orbital floor
blowout fracture. Periorbital muscles and fat can
become herniated and trapped in the fracture
line along the orbital floor causing restriction of
eye movement resulting in diplopia. Intraocular
hemorrhage, such as hyphema, may accompany
the bony injury.
An orbital floor fracture can also be
part of a series of fractures such as
in the case of a zygomatical
maxillary complex fracture.
In this case, the physical findings do
not represent a LeFort III fracture
since there is no contralateral
spectacle hematoma. Since his only
complaints are that of double vision
and a headache, these do not
increase the suspicion for an
intracranial hemorrhage. There is
also no evidence of retinal
detachment at this point.
A 36-year-old female presents with pain and loss of vision in
her right eye, worsening over 3 days. Her PMH is significant
for left TN treated with carbamazepime for 4 years and
depression controlled with sertraline. On neurologic
examination, her right eye vision in 20/400, left is 20/40. She
has a Marcus-Gunn pupil (afferent pupillary defect) on the
right. Her extraocular movements are intact although testing
causes increased pain in the eye. Fundoscopic examination
reveals no abnormality in either eye.
The most likely diagnosis for this patient is
(A) hysteria
(B) carbamazepime toxicity
(C) optic neuritis
(D) amaurosis fugax
The most appropriate next step in the above patient's
management is
(A) psychologic evaluation
(B) serum carbamazepime level and liver function testing
(C) MRI of the brain and possible lumbar puncture
(D) carotid duplex imaging
Answer C.
Explanation: This patient is suffering from optic neuritis. It
has been described as a syndrome in which "the patient can't
see anything and the doctor can't see anything," owing to a
lack of findings on examination. This may lead the examiner
to believe the patient is hysterical.
The patient will have a relative afferent papillary defect
(RAPD or Marcus-Gunn pupil), which is diagnosed with the
swinging light test. The pupils are equal at baseline and
constrict in the light; however, when swinging the light from
the normal eye to the affected eye, the pupils will dilate. This
is due to a relative decrease in afferent stimulation of the
affected eye. Significant monocular vision loss does not occur
without an RAPD.
Optic neuritis is the initial presentation of MS in 15% of
cases, and 50% of patients with MS will develop optic
neuritis at some point in their course. MS is a chronic
demyelinating disease that is usually diagnosed in young
adulthood, and affects women twice as often as men. Its
cause is unknown. The diagnosis is made based on the
history of neurologic symptoms combined with MRI evidence
of lesions explaining the deficits. Lumbar puncture for
elevated IgG index and oligoclonal bands. It can be a
relapsing-remitting or chronic-progressive disease.
TN usually presents in the sixth decade. Its diagnosis in a
young person should prompt a workup for MS.
Amaurosis fugax is usually very transient (minutes), painless,
and the vision loss if often altitudinal, described by the
patient as a shade being pulled over the eye.
How many bones make up the orbit?
(A) 7
(B) 8
(C) 9
(D) 6
(E) 4
Answer A.
Explanation: The following seven bones make up the orbit: frontal,
zygoma, maxilla, palatine, greater and lesser wings of the sphenoid,
lacrimal, and ethmoid bones. Of note, the nasal and temporal bones do
not contribute.
Which of the following statements about
cholesteatoma is true?
(A) It is a malignant tumor.
(B) The primary symptom is tinnitus.
(C) It is caused from eustachian tube dysfunction.
(D) It is a disease of the inner ear.
Answer C.
Explanation: Cholesteatoma is an epidermoid cyst of the
middle ear and/or mastoid, which causes bone destruction
secondary to its expansile nature and through enzymatic
destruction. Cholesteatoma develops as a consequence of
eustachian tube dysfunction and chronic otitis media
secondary to retraction of squamous elements of the
tympanic membrane into the middle ear space. Squamous
epithelium may also migrate into the middle ear via a
perforation. Chronic mastoiditis that fails medical
management or is associated with cholesteatoma is treated
by mastoidectomy.
Laryngoscopic findings after a superior
laryngeal nerve injury include
(A) Ipsilateral vocal cord in a paramedian
position
(B) Ipsilateral vocal cord in a middling
position
(C) Asymmetry of the glottic opening
(D) Normal examination
Answer C.
Explanation: Superior laryngeal nerve injury is less
debilitating, providing the patient's profession is not
related to their vocal performance, as the common
symptom is loss of projection of the voice. The
glottic aperture is asymmetrical on direct
laryngoscopy and management is based on clinical
observation.
Which of the following statements about acute
suppurative parotitis is NOT correct?
(A) Decreased oral intake is a causative factor
(B) Most patients are older than 70 years of age
(C) Parotitis usually develops during the
postoperative period
(D) Poor oral hygiene is a contributing factor
Answer C.
Explanation: These infections develop in elderly individuals
with poor oral hygiene and limited oral intake. The majority
of infections are caused by staphylococci which invade
Stensen's duct where there is minimal parotid secretion.
Although these infections can occur in the postoperative
period, the majority of cases are not related to an operation.
Diagnosis of chronic sinusitis is best made by
(A) Computed tomography scan
(B) Magnetic resonance imaging
(C) Nuclear medicine scanning
(D) History, physical, and nasal endoscopy
Answer D.
Explanation: Nasal endoscopy is a critical element of the
diagnosis of chronic sinusitis. Anatomic abnormalities, such
as septal deviation, nasal polyps, and purulence may be
observed.
The finding of purulence by nasal endoscopy is diagnostic of
sinusitis, regardless of whether other criteria are met. Pus
found on endoscopic exam may be cultured, and subsequent
antibiotic therapy can be directed accordingly. The spectrum
of bacteria found in chronic sinusitis is highly variable and
includes higher prevalences of polymicrobial infections and
antibiotic-resistant organisms.
Overall, S. aureus, coagulase-negative staphylococci, .gramnegative bacilli, and streptococci are isolated, in addition to
the typical pathogens of acute sinusitis
Which of the following statements
concerning surgery for sleep apnea is true?
(A) Surgery is indicated in all patients.
(B) Most patients improve with time, and
surgery is therefore not indicated.
(C) The majority of patients are treated with
tracheostomy alone.
(D) The most common procedure performed
is correction of soft palate collapse.
Answer D.
Explanation: Sleep disorders represent a continuum from simple snoring
to upper airway resistance syndrome (UARS) to obstructive sleep apnea
(OSA). UARS and OSA are associated with excessive daytime
somnolence and frequent sleep arousals.
In OSA, polysomnogram demonstrates at least 10 episodes of apnea or
hypopnea per hour of sleep. The average number of apneas and
hypopneas per hour can be used to calculate a respiratory disturbance
index (RDI), which, along with oxygen saturation, can be used to grade the
severity of OSA. These episodes occur as a result of collapse of the
pharyngeal soft tissues during sleep. In adults, it should be noted that in
addition to tonsil size, factors such as tongue size and body mass index
are significant predictors of OSA. Other anatomic findings associated with
OSA include obese neck, retrognathia, low hyoid bone, and enlarged soft
palate.
Answer D.
Surgery should be considered after failure of more conservative
measures, such as weight loss, elimination of alcohol use, and continuous
positive airway pressure, and should be tailored to the particular
patient's pattern of obstruction.
In children, surgical management typically involves tonsillectomy and/or
adenoidectomy, because the disorder is usually caused by hypertrophy of
these structures.
In adults, uvulopalatoplasty is frequently performed to alleviate softpalate collapse and is the most common operation performed for sleepdisordered breathing. Multiple techniques have been described for this.
Tongue base reduction, tongue advancement, hyoid suspension, and a
variety of maxillomandibular advancement procedures also have been
described with varying success. Adults with significant nasal obstruction
may benefit from septoplasty or sinus surgery.
Patients with severe OSA (RDI >40, lowest nocturnal oxygen saturation
<70%) and unfavorable anatomy or comorbid pulmonary disease may
require tracheotomy.
Trauma of the auricle of the ear with hematoma
formation
(A) Requires transcartilage sutures for
approximation
(B) Requires bolstering for most injuries
(C) Can be treated conservatively with dressings
only
(D) Aggressive débridement is essential
Answer B.
Explanation: With laceration of the auricle, key structures
such as the helical rim and antihelix must be carefully
aligned. These injuries must be repaired such that the
cartilage is covered. The principles of auricular repair are
predicated on the fact that the cartilage has no intrinsic
blood supply and is thus susceptible to ischemic necrosis
following trauma. The suture should be passed through the
perichondrium, while placement though the cartilage itself
should be avoided. Auricular hematomas should be drained
promptly, with placement of a bolster as a pressure
dressing. A pressure dressing is frequently advocated after
closure of an ear laceration. It also deserves note that the
surgeon must avoid the temptation to perform aggressive
débridement after injuries to the eyelid or auricle. Given the
rich vascular supply to the face and neck, many soft-tissue
components that appear devitalized will indeed survive.
Le Fort II fracture entails injuries to all of the
following EXCEPT
(A) Medial wall of the orbit
(B) Alveolus
(C) Zygomaticomaxillary articulation
(D) Nasofrontal buttress
(E) Mandible
Answer E.
Explanation: Le Fort I fractures occur
transversely across the alveolus, above the
level of the teeth apices. In a pure Le Fort I
fracture, the palatal vault is mobile while the
nasal pyramid and orbital rims are stable.
The Le Fort II fracture extends through the
nasofrontal buttress, medial wall of the orbit,
across the infraorbital rim, and through the
zygomaticomaxillary articulation. The nasal
dorsum, palate, and medial part of the
infraorbital rim are mobile.
The Le Fort III fracture is also known as
craniofacial disjunction. The entire face is
mobile from the cranium.
In reality, fractures reflect a combination of
these three types.
Which of the following is the preferred treatment of
tracheal stenosis after prolonged intubation?
(A) Observation
(B) Balloon dilatation
(C) Laser ablation of scar
(D) Resection and primary anastomosis
Answer D.
Explanation: The treatment of tracheal stenosis is resection and primary
anastomosis. In nearly all postintubation injuries the injury is transmural,
and significant portions of the cartilaginous structural support are
destroyed. Measures such as laser ablation are temporizing. In the early
phase of evaluating patients, dilatation using a rigid bronchoscope is
useful to gain immediate dyspnea relief and to fully assess the lesion as
well as its length, position, and relation to the vocal cords. Rarely if ever
is a tracheostomy necessary. For patients unable to tolerate general
anesthesia because of comorbidities, internal stents, typically silicone T
tubes, are useful. Wire mesh stents should not be used, given their
known propensity to erode through the wall of the airway.
The most common
branchial cleft fistula
originates from the
(A) 1st branchial cleft
(B) 2nd branchial cleft
(C) 3rd branchial cleft
(D) 4th branchial cleft
Answer B.
Explanation: Paired branchial clefts and arches develop early
in the fourth gestational week. The first cleft and the first,
second, third, and fourth pouches give rise to adult organs.
The embryologic communication between the pharynx and
the external surface may persist as a fistula. A fistula is seen
most commonly with the second branchial cleft, which
normally disappears, and extends from the anterior border of
the sternocleidomastoid muscle superiorly, inward through
the bifurcation of the carotid artery, and enters the
posterolateral pharynx just below the tonsillar fossa. The
branchial cleft remnants may contain small pieces of cartilage
and cysts, but internal fistulas are rare. A second branchial
cleft sinus is suspected when clear fluid is noted draining
from the external opening of the tract at the anterior border
of the lower third of the sternocleidomastoid muscle. Rarely,
branchial cleft anomalies occur in association with biliary
atresia and congenital cardiac anomalies, an association that
is referred to as Goldenhar's complex.
The most common area of the mandible to be
fractured is the
(A) Condyle
(B) Ramus
(C) Angle
(D) Body
Answer A.
Three days after an accident in which a 25-year-old
woman suffers a maxillary and mandibular fracture,
she develops facial nerve palsy with oral
incompetence and slurred speech. The facial nerve
problem should be managed by
(A) Facial nerve graft
(B) Facial nerve suture
(C) Nonoperative management
(D) Transfer of part of the masseter muscle to the
oral commissure
Answer C.
Explanation: When facial nerve palsy is incomplete or late in
appearance, the nerve injury is partial. With observation, the
palsy will regress over time, and intervention is not required.
The operative techniques listed may be necessary with a
complete nerve injury.
Which of the following is the best
treatment of a septal hematoma
in a patient with a nasal fracture?
(A) Observation
(B) Aspiration of the hematoma
(C) Closed reduction of the
fracture and aspiration of the
hematoma
(D) Operative repair of the
fracture
Answer B.
Explanation: The nose is the most commonly fractured facial
region. The nose is either laterally or posteriorly displaced, and the
fracture may involve the cartilaginous septum, or both the nasal
bones and septum. Patients commonly present with swelling, nasal
deformity, epistaxis, septal deviation, and/or crepitus on palpation.
Intranasal inspection should be performed, and if a septal
hematoma is noted, it should be percutaneously drained. Diagnosis
by computed tomography (CT) scan is not obligatory but is
implemented to rule out other injuries. Immediate treatment
consists of reduction of both the pyramid and septum, followed by
nasal splinting. In spite of early reduction, there is usually a
residual deformity or deviations, which will require formal
rhinoplasty in an elective setting after swelling and bruising have
subsided.
The treatment of choice for cystic hygromas is
(A) Observation
(B) Antibiotics
(C) Intralesional sclerotherapy
(D) Surgical excision
Answer D.
Explanation: The diagnosis of cystic hygroma by prenatal
ultrasound (US) before 30 weeks' gestation has detected a "hidden
mortality," as well as a high incidence of associated anomalies,
including abnormal karyotypes and hydrops fetalis. Occasionally,
very large lesions can cause obstruction of the fetal airway. Such
obstruction can result in the development of polyhydramnios by
impairing the ability of the fetus to swallow amniotic fluid. In these
circumstances, the airway is usually markedly distorted, which can
result in immediate airway obstruction unless the airway is secured
at the time of delivery. Orotracheal intubation or urgent emergency
tracheostomy while the infant remains attached to the placenta, the
ex utero intrapartum technique (EXIT) procedure, may be
necessary to secure the airway
A 2-year-old child swallows a short straight pin and is
brought to the emergency room (ER) by his parents. On
examination, he is alert and able to control his secretions
(i.e., saliva). He has not experienced any respiratory distress
and is afebrile. What is the appropriate course of action?
(A) see the child in the clinic again in 10 days
(B) perform endoscopy if the pin is found in the stomach or
esophagus on x-ray
(C) perform endoscopy whether or not a pin is seen on x-ray
(D) admit the child for observation and daily abdominal plain
films until the pin is passed in the stool
(E) counsel the parents to strain the child's stool and feed
him a high-roughage diet if the pin is radiographically
identified in the stomach
Answer E.
Explanation: Young children make up the majority of patients suffering
from foreign body aspiration: children under 3 account for between 70
and 80% of all foreign body aspirations. Children in this age group tend to
explore with their mouths. Another factor is the lack of development of
molars for grinding and lack of maturity of swallowing and airway
protection processes. Boys outweigh girls by 2:1 in frequency. Whereas
the most common airway foreign body is vegetable matter, esophageal
foreign bodies are coins in 75% of cases. Others may include disc
batteries, screws, tacks, nails, and other hardware items. Increasing in
frequency are toy plastic parts.
The esophagus has four layers: the
mucosa, submucosa, inner circular layer
of muscle, and outer longitudinal layer
of smooth muscle. The upper 5 cm are
skeletal muscle, the upper midsection is
an overlap of striated (skeletal) and
smooth muscle, and the lower half is
smooth muscle. Because there is no
serosa, the esophagus is relatively more
prone to perforation.
There are four anatomic narrowings in
the esophagus: the cricopharyngeus
muscle, aortic crossing, left mainstem
bronchus crossing, and the diaphragm.
The most common area for an
esophageal foreign body to lodge is at
the level of the cricopharyngeus or at
C6. If it lodges elsewhere, investigation
for another congenital anatomic
disorder of the esophagus is warranted.
The signs and symptoms of esophageal foreign body aspiration are
dyspnea or airway distress, drooling, and dysphagia. The wall between
the anterior esophagus and posterior trachea is very compliant and if a
large foreign body is engaged here it can compress the airway from
behind. Any evidence of fever, tachycardia, tachypnea, and increasing
pain should arouse suspicion for esophageal perforation and possible
mediastinal emphysema or retropharyngeal abscess.
Typically, small sharp objects pass spontaneously and thus, this type of
ingestion can be treated conservatively. Objects that require immediate
removal include disc batteries or any ingestions with airway symptoms.
Disc batteries can cause esophageal perforation within 8–12 h of
ingestion, but if radiography reveal they have passed into the stomach,
these ingestions can be treated more conservatively. Coins less than 20
mm in diameter (dimes, pennies) can pass spontaneously. Other objects
that are high risk for causing perforation are long straight pins, chicken
and fish bones, and toothpicks.
Initial workup for any foreign body ingestion are posterior to anterior (PA)
and lateral chest x-rays. The safest method of extraction of esophageal
foreign bodies is a controlled situation with a protected airway under
general anesthesia.
A 19-year-old woman presents to the ER with few days
history of fever and pain in the submandibular region. She
says that over the last several hours she has been having
more trouble speaking with pain in her tongue and is afraid
to lie down. On oral examination, you see that the floor of
mouth is indurated and swollen and very tender. The patient
has very poor dentition but you do not appreciate an
abscess. Her submandibular and submental regions are also
tender and indurated with some fluctuance. What entity in
the differential diagnosis are you most worried about?
(A) Vincent's angina
(B) Bezold's abscess
(C) Ludwig's angina
(D) a retropharyngeal abscess
(E) submandibular and sublingual gland sialadenitis
Answer C.
Explanation: This scenario describes a neck space infection with
abscess. Historically these types of infections were caused by pharyngeal
or tonsillar infections with involvement of the PPC, but since the advent of
antibiotics, these infections are treated early in their course. Most
contemporary adult neck space abscesses are caused by odontogenic or
salivary gland infections, although tonsillar and pharyngeal infections still
account for the majority of pediatric neck space infections. Other etiologies
include preexisting congenital anomalies (branchial cleft sinuses and the
like), trauma, upper respiratory tract infections, iatrogenic causes, or
spread from a superficial infection.
The patient above is exhibiting signs of a submandibular space infection
which has progressed. The majority of these are of odontogenic source,
especially infections of the second and third molars because the roots of
these teeth lie at (second molar) or below the mylohyoid line. The
mylohyoid line separates the sublingual and submandibular spaces. If this
infection goes untreated it rapidly progresses to a gangrenous cellulitis
with brawny induration involving bilateral sublingual, submental (between
anterior bellies of the digastric muscles and between the mylohyoid muscle
and skin), and submandibular spaces.
The clinical presentation is marked by drooling, severe pain, trismus,
dysphagia, and respiratory distress. Because of floor of mouth swelling
and induration, the tongue is compressed against the palate, thereby
obstructing the oral airway. Ludwig's angina is the deep neck space
infection which is most associated with the need for tracheostomy.
The typical microorganisms involved are oral flora, such as
Peptostreptococcus, Streptococcus pyogenes, Fusobacterium as well as
Bacteroides melaninogenicus and Staphylococcus aureus. Penicillin
remains the drug of choice but any antibiotic with a similar spectrum (i.e.,
clindamycin, first generation cephalosporins) is usually adequate.
Most neck space infections in the abscess stage require surgical drainage.
Vincent's angina, also known as trench mouth, is an acute necrotizing
ulcerative gingivitis secondary to a mixed anaerobic infection. Patients
present with malodorous breath, drooling and gingival bleeding; penicillin
and adequate oral hygiene are the treatments. Bezold's abscess refers to
a postauricular abscess secondary to mastoiditis. A retropharyngeal
abscess can also present with symptoms of dysphagia and odynophagia,
snoring, noisy breathing and cervical adenopathy, but airway obstruction is
less common. Retropharyngeal infections are more common in children as
lymph nodes (which are the typical source) regress or atrophy by the age
of 4 or 5.
A 14-year-old male is involved in a dirt bike accident in which he suffers a
"clothesline" injury. On examination in the ER you see a 7 cm laceration
in the anterior neck, subcutaneous emphysema, and a hematoma which
does not appear to be expanding. He is unable to lay flat and has a
muffled voice. On flexible laryngoscopy, you see diffuse but mild edema
of the supraglottis and glottis, reduced vocal cord abduction, and bloody
secretions in the subglottis. Initial management of this patient would
involve
(A) nasal intubation, laryngeal and cervical spine CT, exploration and
repair with intraoperative tracheotomy
(B) tracheostomy under local anesthesia, cervical spine series,
endoscopy, exploration and repair
(C) percutaneous tracheostomy, cervical spine series, exploration and
repair with stenting
(D) oral intubation, laryngeal and cervical spine CT, endoscopy,
exploration and repair
(E) tracheostomy under general anesthesia, CT of the larynx and cervical
spine, endoscopy, exploration and repair with stenting over a T-tube
Answer B.
Explanation: External laryngeal trauma is diagnosed on the basis of
history and physical findings. A patient who presents with evidence of
anterior neck trauma should be assumed to have upper airway trauma.
This compounded with subcutaneous emphysema, voice changes, and
orthopnea should arouse suspicion for disruption of the larynx or trachea.
As in any trauma situation, the "ABCs" come first: airway, breathing and
circulation. Although on fiberoptic examination this patient had "mild
edema" it is presumable early after the trauma and the entire injury may
have not evolved. There is potential for worsening of the edema and
bleeding in the next 8–12 h. As a result, an awake tracheostomy is the best
option. The addition of general anesthesia in this situation may cause
laryngospasm and resultant complete airway obstruction. In addition,
"clothesline" injuries are high risk for being associated with laryngotracheal
separation. Any situation in which this is considered precludes oral or nasal
intubation as intubation may worsen the existing damage or convert a
partial laryngotracheal or cricotracheal separation into a complete
separation.
The pathophysiology behind blunt trauma to the larynx involves crushing of
the laryngeal skeleton against the cervical spine. There is a shearing effect
between the laryngeal ligaments, the thyroarytenoid (vocalis) muscle, and
the perichondrium of the thyroid and cricoid cartilages. In addition
arytenoid cartilage dislocation or subluxation and recurrent laryngeal nerve
injury via traction or actual transection may occur. The result is mucosal
tears, edema, and hematoma or hemorrhage. A "clothesline" injury can be
associated with bilateral recurrent nerve damage. Any damage to the
cricoid can be particularly devastating as it is the only complete ring of the
airway and is the cornerstone of structural support for the larynx.
Some external laryngeal trauma can be treated conservatively with
medical management. Conditions include: minor edema or hematomas
with intact mucosa, single nondisplaced thyroid cartilage fractures, small
lacerations without exposed cartilage. Medical management would include
elevation of the head of bed with bedrest to reduce edema. Any patient not
meeting the criteria for conservative management proceeds to surgery.
Frequently, the lacerations are used to explore the laryngeal framework
and mucosa. Early intervention is advocated for less scarring and
granulation tissue.
Which of the following is an indication for
tonsillectomy?
(A) Patient's request
(B) Chronic middle ear infection
(C) Three or more infections per year
(D) Missing more than one week of school
per year
Answer C.
Explanation: Tonsillectomy and adenoidectomy are indicated for
chronic or recurrent acute infection and for obstructive hypertrophy.
The American Academy of Otolaryngology–Head and Neck
Surgery Clinical Indicators Compendium (2000) suggests
tonsillectomy after three or more infections per year despite
adequate medical therapy. Some feel that tonsillectomy is indicated
in children who miss 2 or more weeks of school annually secondary
to tonsil infections. Multiple techniques have been described,
including electrocautery, sharp dissection, laser, and
radiofrequency ablation. There is no consensus as to the best
method. In cases of chronic or recurrent infection, surgery is
considered only after failure of medical therapy.
The most likely pathogen to
be involved with supraglottitis
(epiglottitis) is
(A) Streptoccocus pneumonia
(B) H. influenzae
(C) influenza virus
(D) parainfluenza virus
(E) S. aureus
Answer B.
Explanation: Despite the advent and widespread use of the HIB vaccine,
H. influenzae type b still remains the most common cause of epiglottitis.
Historically, the disease was more common in children between ages 2
through 6; however, with vaccine use, the incidence in children has
dropped from 3.5 in 100,000 to 0.6 in 100,000, whereas that in adults has
remained the same or has risen slightly. Other bacteria that are found
commonly include other types of H. influenzae, -hemolytic streptococci,
Staphylococcus, Klebsiellae pneumoniae, Bacteroides melanogenicus, and
Mycobacterium tuberculosis.
The presentation in children is fever, sore throat of a rapid onset with
inspiratory stridor; adults will also complain of odynophagia. The key is
the rapid onset of pain with a paucity of oropharyngeal findings (such as
lack of evidence of acute tonsillitis or peritonsillar abscess). Children may
have trouble handling secretions and may drool and patients in general
may have a muffled or "hot potato" voice all related to edema of the
epiglottis. Patients sit forward and upright in a "sniffing" position to
relieve some of the respiratory obstruction.
Diagnosis is based chiefly on history and physical examination. Though a
classic "thumbprint" sign of the epiglottis on lateral neck x-ray has been
described in the setting of supraglottitis, the sensitivity of lateral neck
films is on the order of 40% and the specificity around 75%. If there is any
suspicion of this process, final diagnosis via laryngoscopy should be
performed in a controlled setting in the operation room (OR). The
epiglottis alone is affected in children by appearing beefy red and
edematous, whereas in the adult, all supraglottic tissue is inflamed
appearing and edematous and thus is referred to as supraglottitis.
Attempts should be made to not arouse the patient or make them upset
as this may precipitate complete airway obstruction. Oral intubation
should be performed by the most skilled person available to avoid
trauma and subsequent further reactive edema from the epiglottis.
Preparations should be ready for tracheostomy if needed.
The patient is left intubated for 48–72 h during which time the edema
should have subsided. The patient is placed on broad spectrum
antibiotics that especially have activity against H. influenzae and is kept in
the intensive care unit (ICU).
The patient should be extubated only after direct assessment of the
airway via laryngoscopy and an endotracheal tube leak test to confirm
edema resolution. Although steroids are sometimes employed to
decrease edema, there is no definitive data to advocate their use.
Despite the technological advances of the last two decades, the mortality
rate is still 1.6% for adults.
Epiglottitis should be differentiated from two other respiratory disorders:
viral croup and bacterial trachneitis. Viral croup is found in children less
than 2 years and is associated with parainfluezae and respiratory
syncytial viruses. The stridor is usually biphasic as the inflammation is
subglottic compared to epiglottitis which has an associated inspiratory
stridor. There is a characteristic "seal-barking"cough and patients usually
do not have odynophagia. These patients are treated with humidity,
inhaled steroids and racemic epinephrine as well as antibiotics to prevent
superinfection. Bacterial tracheitis is usually a staphylococcal infection of
the trachea that can occur at any age and patients present with an
expiratory stridor and hoarseness. This is also a serious disorder and
requires ICU care, bronchoscopy with suctioning, and IV antibiotics.
A 12-year-old female presents for evaluation of a neck mass.
Which of the following pairs are correct?
(A) branchial cleft cyst/sinus: most commonly involves the
third branchial cleft remnant
(B) thyroglossal duct cyst (TGDCs): mesodermal remnants
that produce lateral swelling over neck
(C) cystic hygroma: a salivary gland disorder related to a
hypersecretory cyst
(D) torticollis: unilateral shortening of trapezius muscle
(E) medullary thyroid cancer: most common cause of death
in multiple endocrine neoplasia (MEN) 2B
Answer E.
Explanation: Commonly encountered developmental neck abnormalities in
children are of congenital origin yet may not cause problems or be detected
until adulthood. While some of these neck lesions may appear
asymptomatic at birth, they may precipitously become enlarged and
disfiguring as a result of local or regional infection or hemorrhage.
Developmental abnormalities of the branchial apparatus represent a
common source of congenital lateral neck masses. Branchial anomalies may
present as a cyst, sinus, or fistula. Branchial cleft anomalies arise most
commonly (greater than 90%) from the second branchial cleft system. Eight
percent arise from the first branchial anomaly whereas third and fourth
branchial malformations are rare. Usually the second branchial cleft sinus or
fistula presents with drainage from a small pit in the skin just anterior to the
lower third of the sternocleidomastoid muscle. Treatment of choice is
surgical excision due to the risk of infection
Thyroglossal duct cysts (TGDCs) represent the most common head and
neck midline masses in children. It is reported that they account for
about 70% of all congenital neck abnormalities. TGDCs are embryonic
ectodermal rests that can present as midline structures as they follow
the descent along the thyroid gland tract. Normally, the thyroglossal duct
regresses once the thyroid gland reaches the anterior neck. Faulty
thyroid migration or persistence of the thyroglossal duct can lead to the
formation of lingual/ectopic thyroid tissue, pyramidal thyroid lobe, or a
TGDCs. Since TGDCs are attached to the hyoid bone, clinical presentation
typically shows a midline mass that moves with swallowing. Treatment is
based on the Sistrunk procedure in which complete surgical excision of
the cyst and tract up to the base of the tongue including the central
portion of the hyoid bone is preformed.
Lymphatic malformations commonly referred to as cystic
hygromas are developmental abnormalities of the
lymphoid system that occur at sites of lymphatic-venous
connection, most commonly in the posterior neck. The
cysts may become enlarged and disfiguring not only as a
result of infection or hemorrhage but also due to increases
in fluid and endothelial cell growth. Imaging by US, CT, and
MRI (for complex and extensive lesions) is mandated to
determine whether involvement of deeper airway
structures is present. This also gives pertinent clues as to
the planning of the operative approach. When these
lesions are diagnosed prenatally, the overall prognosis is
poorer than those diagnosed after birth. Treatment of
these lesions is primarily surgical but another therapy is
injection sclerotherapy with such agents as bleomycin, OK432, sodium morrhuate, 22.5% glucose, and triamcinolone.
Sclerotherapy is usually reserved for extensive disease or
recurrences.
Torticollis is a deformity characterized by the
unnatural tilted or turned position of the head. The
most common form is due to shortening of the
sternocleidomastoid muscle, although a number of
other conditions can potentially cause torticollis
(cervical hemivertebrae, adenitis, fascitis, and
oculomotor abnormalities). Birth trauma was once
thought to contribute to the cause of torticollis by
injury to the sternocleidomastoid or the spinal
accessory nerve, but this is rarely the case. The
mother or primary physician usually notes the classic
presentation of an otherwise healthy 2–8-week-old
infant who preferentially turns their head to one side.
Compete resolution of untreated torticollis occurs in
50–70% of cases by 6 months of age, but because it
is difficult to predict which infants will develop an
irreversible deformity, a passive range-of-motion
exercise regimen is advocated. In cases that present
with or develop facial hemihypoplasia, surgery to
divide the sternocleidomastoid on the affected side is
indicated
Answer E.
Medullary thryroid cancer can occur sporadically, in association with
MEN types 2A or 2B, or with the familial medullary cancer syndrome. A
mutation in the ret protooncogene in individuals with MEN and the
familial variant predisposes family members (autosomal dominant
inheritance) to the development of medullary thyroid cancer at an early
age. This tumor is the first to develop in MEN children and is the most
common cause of death. In these children, early thyroidectomy is
advocated after the genetic mutation has been confirmed. MEN 2A
children should undergo thyroidectomy prior to 5 years of age; whereas
children of MEN 2B require thyroidectomy prior to 1 year of age due to
the more virulent nature of the disease.
Hyperacute rejection is caused by
A. Preformed antibodies
B. B-cell–generated antidonor antibodies
C. T-cell–mediated allorejection
D. Nonimmune mechanism
Answer A.
Explanation: Hyperacute rejection, which usually occurs within minutes
after the transplanted organ is reperfused, is due to the presence of
preformed antibodies in the recipient, antibodies that are specific to the
donor. These antibodies may be directed against the donor's HLA
antigens or they may be anti-ABO blood group antibodies. Either way,
they bind to the vascular endothelium in the graft and activate the
complement cascade, leading to platelet activation and to diffuse
intravascular coagulation. The result is a swollen, darkened graft, which
undergoes ischemic necrosis. This type of rejection is generally not
reversible, so prevention is key.
Prevention is best done by making sure the graft is ABO-compatible and
by performing a pretransplant cross-match. The cross-match is an in vitro
test that involves mixing the donor's cells with the recipient's serum to
look for evidence of donor cell destruction by recipient antibodies. A
positive cross-match indicates the presence of preformed antibodies in
the recipient that are specific to the donor, thus a high risk of hyperacute
rejection if the transplant is performed.
The mechanism of action of azathioprine is
A. Inhibition of calcineurin
B. Interference with DNA synthesis
C. Binding of FK-506 binding proteins
D. Inhibition of P7056 kinase
Answer B.
Explanation: Azathioprine (AZA) acts late in the immune process,
affecting the cell cycle by interfering with DNA synthesis, thus
suppressing proliferation of activated B and T lymphocytes. AZA is
valuable in preventing the onset of acute rejection, but is not effective in
the treatment of rejection episodes themselves.
Cyclosporine binds with its cytoplasmic receptor protein, cyclophilin,
which subsequently inhibits the activity of calcineurin. Doing so impairs
expression of several critical T-cell activation genes, the most important
being for interleukin-2 (IL-2). As a result, T-cell activation is suppressed.
The metabolism of cyclosporine is via the cytochrome P450 system,
therefore several drug interactions are possible. Inducers of P450 such as
phenytoin decrease blood levels; drugs such as erythromycin, cimetidine,
ketoconazole, and fluconazole increase them.
Tacrolimus, like cyclosporine, is a calcineurin inhibitor and has a very
similar mechanism of action. Cyclosporine acts by binding cyclophilins,
while tacrolimus acts by binding FK506-binding proteins (FKBPs). The
tacrolimus-FKBP complex inhibits the enzyme calcineurin. The net effect
of tacrolimus is to inhibit T-cell function by preventing synthesis of IL-2
and other important cytokines.
Answer B.
Sirolimus (previously known as rapamycin) is structurally similar to
tacrolimus and binds to the same immunophilin (FKBP). Unlike
tacrolimus, it does not affect calcineurin activity, and therefore does not
block the calcium-dependent activation of cytokine genes. Rather, the
active complex binds so-called target of rapamycin (TOR) proteins
resulting in inhibition of P7056 kinase (an enzyme linked to cell division).
The net result is to prevent progression from the G1 to the S phase of the
cell cycle, halting cell division.
Mycophenolate mofetil works by inhibiting inosine monophosphate
dehydrogenase, which is a crucial, rate-limiting enzyme in de novo
synthesis of purines. Specifically, this enzyme catalyzes the formation of
guanosine nucleotides from inosine. Many cells have a salvage pathway
and therefore can bypass this need for guanosine nucleotide synthesis by
the de novo pathway. Activated lymphocytes, however, do not possess
this salvage pathway and require de novo synthesis for clonal expansion.
The net result is a selective, reversible antiproliferative effect on T and B
lymphocytes.
In the prevention of graft rejection,
cyclosporine
A. Blocks transcription of interleukin-1 (IL-1)
and tumor necrosis factor- (TNF-)
B. Inhibits lymphocyte nucleic acid
metabolism
C. Results in rapid decrease in the number of
circulatory T lymphocytes
D. Selectively inhibits T-cell activation
Answer D.
Explanation: There are a number of different agents used to control graft
rejection, and they function in different ways. Cyclosporine, the mainstay of
immunosuppression, selectively inhibits T-cell activation. Corticosteroids
block the transcription of IL-1 and TNF-. Azathioprine inhibits lymphocyte
nucleic acid metabolism. Mycophenolate mofetil inhibits RNA and DNA
synthesis. OKT3 results in a rapid decrease in circulatory T lymphocytes.
Lymphoceles occur how long after a renal
transplant?
A. Within 48 h
B. ~1 week after surgery
C. 2–4 weeks after surgery
D. 3 months after surgery
Answer C.
Explanation: The reported incidence of lymphoceles (fluid collections of
lymph that generally result from cut lymphatic vessels in the recipient) is
0.6 to 18%. Lymphoceles usually do not occur until at least two weeks
posttransplant. Symptoms are generally related to the mass effect and
compression of nearby structures (e.g., ureter, iliac vein, allograft renal
artery), and patients develop hypertension, unilateral leg swelling on the
side of the transplant, and elevated serum creatinine. Ultrasound is used to
confirm a fluid collection, although percutaneous aspiration may be
necessary to exclude presence of other collections such as urinomas,
hematomas, or abscesses. The standard surgical treatment is creation of a
peritoneal window to allow for drainage of the lymphatic fluid into the
peritoneal cavity where it can be absorbed. Either a laparoscopic or an
open approach may be used. Another option is percutaneous insertion of a
drainage catheter, with or without sclerotherapy; however, it is associated
with some risk of recurrence or infection.
Which of the following is NOT a side effect of
cyclosporine?
A. Interstitial fibrosis of the renal parenchyma
B. Gingival hyperplasia
C. Headache
D. Pancreatitis
Answer D
Explanation: Adverse effects of cyclosporine can be classified as
renal or nonrenal. Nephrotoxicity is the most important and
troubling adverse effect of cyclosporine. Cyclosporine has a
vasoconstrictor effect on the renal vasculature. This
vasoconstriction (likely a transient, reversible, and dose-dependent
phenomenon) may cause early posttransplant graft dysfunction or
may exaggerate existing poor graft function. Also, long-term
cyclosporine use may result in interstitial fibrosis of the renal
parenchyma, coupled with arteriolar lesions. The exact mechanism
is unknown, but renal failure may eventually result.
A number of nonrenal side effects may also be seen with the use of
cyclosporine. Cosmetic complications, most commonly hirsutism
and gingival hyperplasia, may result in considerable distress,
possibly leading to noncompliant behavior, especially in
adolescents and women. Several neurologic complications,
including headaches, tremor, and seizures, also have been
reported. Other nonrenal side effects include hyperlipidemia,
hepatotoxicity, and hyperuricemia.
The 5-year graft survival rate after renal
transplantation is
A. 35–40%
B. 50–55%
C. 75–80%
D. 90–95%
Answer C.
Explanation: The incidence of acute rejection has declined steadily since
the early 1990s. Most centers now report acute rejection rates of 10 to
20% at 1 year posttransplant. This decline has been a major factor in the
improvement in graft survival rates, which are now about 75 to 80% at 5
years and 60 to 65% at 10 years posttransplant for all kidney recipients.
Currently, the most common cause of graft loss is recipient death (usually
from cardiovascular causes) with a functioning graft. The second most
common cause is chronic allograft nephropathy. Characterized by a slow,
unrelenting deterioration of graft function, it likely has multiple causes (both
immunologic and nonimmunologic). The graft failure rate due to surgical
technique has remained at about 2%.
All of the following are absolute contraindications in
considering a candidate for orthotopic cardiac
transplantation EXCEPT
A. Active infection
B. Age over 65 years
C. History of medical noncompliance
D. Severe renal insufficiency
Answer A.
Explanation: Active infection is considered a potentially reversible
contraindication to cardiac transplantation. The other conditions listed are
absolute contraindications to orthotopic cardiac transplantation.
Heterotopic cardiac transplantation, in which the patient's right heart
continues to work against the pulmonary hypertension while the donor
heart supplies systemic circulation, is used for a certain number of patients
with pulmonary hypertension.
After completion of the vascular
anastomoses, drainage of a transplanted
pancreas is accomplished by anastomosis to
A. Right colon
B. Left colon
C. Duodenum
D. Bladder or small bowel
Answer D
Explanation: Once the pancreas is revascularized, a drainage procedure must be
performed to handle the pancreatic exocrine secretions. Options include
anastomosing the donor duodenum to the recipient bladder or to the small bowel,
with the small bowel either in continuity or connected to a Roux-en-Y limb. Some
centers always use enteric drainage, others always use bladder drainage, and
others tailor the approach according to the recipient category. Both enteric drainage
and bladder drainage now have a relatively low surgical risk. The main advantage of
bladder drainage is the ability to directly measure enzyme activity in the pancreatic
graft exocrine secretions by measuring the amount of amylase in the urine. A
decrease in urine amylase is a sensitive marker for rejection, even though it is not
entirely specific.
Absolute contraindications for donation of a
heart include all of the following EXCEPT
A. Carbon monoxide-hemoglobin level >20%
B. Prolonged cardiac arrest
C. Prolonged high-dopamine requirement
D. Significant smoking history
Answer C.
Explanation: The use of high doses of dopamine for more than 24 h
before death is a relative contraindication to transplantation of the heart.
The other listed items are all absolute contraindications to cardiac
donation. Severe structural heart disease and human immunodeficiency
virus seropositivity are other absolute contraindications.
The most common cause of renal failure in
the United States is
A. Chronic glomerulonephritis
B. Chronic pyelonephritis
C. Diabetes mellitus
D. Obstructive uropathy
Answer C.
Explanation: Because the life expectancy of patients with diabetes
mellitus has dramatically lengthened by appropriate use of insulin,
diabetes is now the leading cause of renal failure and contributes to
blindness, neuropathies, and early atherosclerosis. These problems have
led to the continued interest in the possibility of pancreatic transplantation
as a form of disease control.
Immunologic rejection is mediated by the
recipient's
A. Eosinophils
B. Lymphocytes
C. Neutrophils
D. Plasma cells
Answer B.
Explanation: Early work in the transplantation field showed that graft
rejection was mediated by the recipient's white blood cells. Refinement of
the techniques involved demonstrated that the lymphocytes played the
major role in this phenomenon. The development of antilymphocyte serum
was an early step in controlling the rejection process.
About Caroli's disease, mark the correct answer(s).
A. It is characterized by intrahepatic bile duct
atresia.
B. Abdominal mass and weight loss are the most
common initial symptoms.
C. It is a developmental anomaly of the ductal plate
characterized by saccular dilatations of the large bile
ducts.
D. It is more commonly seen in adult females.
E. It is a risk factor for the development of
cystadenocarcinoma of the bile duct.
Answer C.
Which of the following is not a complication
of Caroli's disease?
A. stone formation
B. recurrent cholangitis
C. septicemia
D. cholangiocarcinoma
E. amyloidosis
F. renal disorders
Answer F.
Which of the following radiologic studies is
not recommended for the diagnosis of
Caroli's disease?
A. magnetic resonance imaging (MRI)
abdomen
B. CT scan abdomen
C. abdominal US
D. HIDA scan
E. ERCP
Answer D.
Explanation: Caroli's disease is an abnormal development of the
intrahepatic bile ducts without an obstructive cause, characterized by
saccular dilatations, resembling a picture of multiple cyst-like structures of
varying size. Two types have been described: a type with bile duct
abnormalities alone and a type with bile duct abnormality combined with
periportal fibrosis, similar to congenital hepatic fibrosis. This combined type
is also known as Caroli's syndrome and has been reported more frequently
than the pure type, or Caroli's disease.
Caroli's disease is anatomically characterized for saccular dilatations of the
bile ducts more frequently seen in the left side of the liver. In 30–40% of
the cases this are confined to one segment of one side of the liver.
Bilateral abnormalities are more common in the second type: Caroli's
syndrome.
The most common complications are cholangitis, septicemia, amyloidosis,
and cholangiocarcinoma (7–10% of patients). Caroli's syndrome is
associated with renal disorders such as renal cysts and nephrospongiosis
seen in 30–40% of patients. These disorders have not been seen in
Caroli's disease.
The diagnosis is made by radiologic studies such as US, CT scan, ERCP,
MRI where saccular or cystically dilated intrahepatic ducts are seen.
Surgical treatment is indicated in order to reduce the risk of recurrent
cholangitis, biliary cirrhosis, or cholangiocarcinoma. Hepatic lobectomy is
indicated for localized bile duct abnormalities (Caroli's disease), while liver
transplant should be considered in selected patients with generalized
disease or concomitant liver fibrosis and portal hypertension (Caroli's
syndrome).
Which of the following is not considered a
risk factor for cholangiocarcinoma
development?
A. primary sclerosing cholangitis
B. Caroli's disease
C. choledocal cyst
D. biliary atresia
E. hepatolithiasis
Answer D.
Explanation: Cholangiocarcinoma is an uncommon cancer found in 0.01–0.2%
of all autopsies. The majority of the patients are older than 65-year-old with a
male predominance.The etiology is unknown, but several predisposing
conditions have been identified:
1. Primary sclerosing cholangitis have a 6–30% chance of developing it and
10–30% of patients who undergo liver transplant for PSC have an occult
cholangiocarcinoma.
2. Congenital biliary cystic disease (Caroli's disease, choledochal cyst) have a
15–20% chance of cancer.
3. Hepatolithiasis (recurrent pyogenic cholangitiohepatitis or oriental
cholangiohepatitis), prevalent in Japan, secondary to chronic portal
bacteremia and portal phlebitis which may give rise to intrahepatic pigment
stone formation.
4. Biliary parasites, especially Clonorchis sinensis and Opisthorchis viverrini,
are associated with an increased risk.
5. Carcinogens such as thorium, radon, nitrosamines, dioxin, and asbestos.
Different classifications have been used to describe this tumor. Anatomically
they can be divided in intra- and extrahepatic, where 94% of the
cholangiocarcinomas are extrahepatics and perihiliar (67%).
Answer D.
The most commonly used classification is the
Bismuth classification, which describes the tumor
in perspective to the bile duct bifurcation and has
direct implication on the surgical strategy.
Bismuth 1: Tumor below hepatic bifurcation and
can be treated with bile duct resection alone.
Bismuth 2: Tumor that reaches the bifurcation.
They usually require a caudate lobe resection, in
addition to bile duct resection.
Bismuth 3a: They may reach the second
intrahepatic division of the right main duct.
Bismuth 3b: They extend to the left main bile
duct. Both require either a right or left
hemihepatectomy with bile duct resection.
Bismuth 4: Affects both main bile ducts. Surgery
is not an option in this type of location.
Required laboratory tests in evaluation of a
patient under consideration for heart
transplantation include all of the following
EXCEPT
A. Blood type
B. Cardiac catheterization
C. Complete blood count
D. Prothrombin time and activated partial
thromboplastin time
Answer B.
Explanation: Cardiac catheterization may be indicated in some patients to
evaluate cardiac function. The other tests are required in any patient under
consideration for cardiac transplantation.
All of the following conditions in a potential
donor are absolute contraindications to the
use of a kidney for transplantation EXCEPT
A. Age older than 70 years
B. Chronic renal insufficiency
C. Long-standing hypertension
D. Presence of hepatitis C
Answer D.
Explanation: Cadaveric kidneys make up 75% of all donor kidneys, and
the demand far exceeds the supply. For this reason, donor criteria have
been liberalized in recent years. Advanced age, chronic renal insufficiency,
intravenous drug abuse, and long-standing hypertension remain absolute
contraindications. Human immunodeficiency virus seropositivity and the
presence of surface antigens against hepatitis B are also absolute
contraindications. Although there is risk associated with using a kidney
from a donor with evidence of hepatitis C, this condition is not considered
an absolute contraindication to kidney use.
Absolute contraindications to renal
transplantation for a patient with chronic
renal failure include all of the following
EXCEPT
A. Chronic active hepatitis
B. Human immunodeficiency virus infection
C. Recent operation of cancer of the colon
D. Sickle cell disease
Answer D.
Explanation: Sickle cell disease is a relative contraindication to renal
transplantation because of the associated high incidence of recurrence.
The other listed conditions are considered absolute contraindications
because of the patient's generally poor health prognosis.
The single most important factor in
determining whether to perform a transplant
between a specific donor and recipient is
A. Mixed lymphocyte culture assays of the
donor and recipient
B. HLA types of the donor and recipient
C. ABO blood types of the donor and
recipient
D. Peripheral T-cell count of the recipient
Answer C.
Explanation: Although mixed lymphocyte culture assays and HLA typing
of the donor and recipient to determine compatibility have been shown to
enhance long-term graft survival, immediate graft function has been
correlated to the absence of the presensitized state. This presensitization
can be with respect to lymphocytotoxic antibodies or preformed
isoagglutinins. ABO compatibility is essential in renal and cardiac
transplantation because incompatibility leads to prompt destruction of the
transplanted organ. In liver transplantation, the presensitized state is of
less importance, but diminished graft survival has been demonstrated in
ABO-incompatible combinations.
A 24-year-old woman is admitted to the intensive care unit for sudden
collapse with progressive neurologic deficits. A computed tomography
scan reveals an intracranial tumor with evidence of acute hemorrhage.
Emergent craniotomy is done for decompression, and tissue obtained
reveals high-grade malignant astrocytoma. On postoperative day 1, the
patient is placed on large doses of phenobarbital for seizure activity but
continues to deteriorate. On day 3, she requires dopamine support at 10
mg/kg/min to maintain a systolic blood pressure of 90 mm Hg. She
develops diabetes insipidus, and her urine output is adequate, although
her creatinine rises to 1.5 mg/dL and the blood urea nitrogen (BUN) is 40
mg/dL. A urine culture from a Foley specimen yields Escherichia coli at
100,000/mL. On day 4, she becomes unresponsive, without evidence of
cortical or brainstem function. An electroencephalogram (EEG) is
isoelectric. Which of the following is an absolute contraindication for
consideration of this woman as a potential organ donor?
A. Presence of high-grade intracranial malignancy
B. Requirement of pressor support
C. Elevated BUN and creatinine
D. Presence of supratherapeutic phenobarbital levels
Answer D.
Explanation: The presence of phenobarbital, narcotics or alcohol, or of
hypothermia is a contraindication to organ donation, even with an
isoelectric EEG. This is because these factors may suppress spontaneous
electric activity of the brain. Intracranial tumors are not considered a
contraindication, mainly because of their lack of systemic metastasis.
Other malignancies do contraindicate donation. The need for pressor is not
necessarily a contraindication, especially if these drugs are used in the
terminal period to maintain blood pressure and urine output. Elevations of
BUN and creatinine are not uncommon, especially in the face of diabetes
insipidus with prerenal azotemia. Adequate urine output is the most
important factor in consideration of renal organ donation. Lower urinary
tract infection caused by instrumentation is not a contraindication to
donation of kidneys, whereas systemic or peritoneal sepsis is.
All of the following are side effects of
cyclosporine A administration for prevention
of organ rejection EXCEPT
A. Hepatotoxicity
B. Hirsutism
C. Tremor
D. Bone marrow depression
Answer D.
Explanation: Bone marrow depression has often been seen with
azathioprine but is not seen in patients on cyclosporine. Hepatotoxicity,
hirsutism, tremor, and nephrotoxicity are complications of prolonged use of
cyclosporine A. Nephrotoxicity is the most clinically important and most
frequently seen side effect and may limit the drug's use in some patients.
Currently, which of the following infectious
illnesses is most likely to compromise
patients after renal transplantation?
A. Coli sepsis
B. Pneumococcal sepsis
C. Candidiasis
D. Cytomegalovirus sepsis
Answer D.
Explanation: Immunosuppression for transplantation increases the risk for
all types of infection. Use of cyclosporine, along with broad-spectrum
antibiotics, has reduced the incidence of bacterial infections. Most serious
posttransplant infections arise when rejection is being treated, and in the
past, these led to high mortality. Both Candida and Aspergillus infections
can occur but are relatively rare compared with viral infection.
Cytomegalovirus (CMV) can produce a spectrum of illness characterized
by fever, neutropenia, arthralgias, malaise, gastrointestinal ulcerations, and
decreased renal function. CMV itself produces a state of
immunosuppression, and many serious infections are superinfections in
patients already experiencing CMV infections. Treatment of CMV sepsis
includes decreasing immunosuppression and administering ganciclovir, a
new antiviral drug.
Postoperative indicators of primary
nonfunction of a liver allograft include all of
the following EXCEPT
A. Hypokalemia
B. Hypoglycemia
C. Elevated prothrombin time
D. Alkalosis
Answer A.
Explanation: Primary graft failure is a very serious complication. The
patient decompensates quickly, and urgent transplantation is indicated.
Severe central nervous system changes, with acid-base changes (early
alkalosis due to inability to metabolize citrate and acidosis as a terminal
event), hyperkalemia, coagulopathy, hypoglycemia, and oliguria are often
terminal events of this acute hepatic decompensation.
An absolute contraindication to cardiac
transplantation is
(A) Active peptic ulcer disease
(B) Age older than 60 years
(C) Fixed pulmonary vascular resistance
(D) Heavy cigarette smoking
Answer C.
Explanation: Although the other three items are relative contraindications
to cardiac transplantation, fixed pulmonary vascular resistance is the only
absolute contraindication among those listed. A heart accustomed to low
pulmonary artery pressure and resistance will fail immediately if placed in a
recipient with fixed pulmonary vascular resistance.
A 45-year-old female underwent a kidney transplant 6
months ago and has been taking cyclosporine and steroids.
She developed cholelithiasis and requires a laparoscopic
cholecystectomy. She wants to know if her chance to have a
wound infection is increased. The best answer to her
question is
A. No, steroids and cyclosporine do not increase the chance
to have wound infection when used chronically
B. Yes, because of inhibition of collagen synthesis and
fibroblast proliferation
C. Yes, because of persistent vasoconstriction and hypoxia
D. Yes, because of structural nuclear changes and decreased
DNA synthesis
E. Yes, mainly because of cyclosporine, which blocks IL-2 and
decrease migration of macrophages
Answer B.
Explanation: During the past two decades the survival rate of solid organ
recipients has improved dramatically. The major factor in improved clinical
outcome is the decline in death secondary to infection. Currently, 1-year
mortality caused by infection has decreased to less than 5% for renal
transplant patients. Better immunosuppression and understanding by the
clinician of drug pharmacodynamics and pharmacokinetics are responsible
for decrease morbidity and mortality after solid organ transplantation.
Steroids reduce the inflammatory process blocking transcription of cytokine
genes (especially IL-1) leading to nonspecific inhibition of T lymphocytes
and macrophages. It is well documented that steroids decrease fibroblast
migration and collagen synthesis. Topical steroids also inhibit wound
healing.Cyclosporine does not significantly affect hydroxyproline content
and macrophages migration, although there is some evidence that
cyclosporine impairs wound healing. Studies in rats have shown that
activin- expression and matrix metalloproteinases (MMPs) activity by
fibroblast is reduced.
The best method of monitoring the
development of acute rejection in a patient
after cardiac transplantation is
A. Dipyridamole thallium study
B. Electrocardiogram
C. Endomyocardial biopsy
D. Ultrasound examination of the heart
Answer C.
Explanation: It would be desirable to follow possible rejection by some
noninvasive procedure, but none has given timely results.
Endomyocardial biopsies allow rejection to be diagnosed before
significant organ damage and dysfunction occur.
All of the following examination findings are
consistent with a diagnosis of brain death
except:
A. dilated and nonreactive pupils
B. absent oculocephalic reflex
C. extensor (decerebrate) posturing
D. absent gag reflex
Answer C.
Explanation: There are two reasons to declare that a patient is brain
dead. The first is to allow for organ donation, and the second is to allow for
removal of life support mechanisms once it is deemed that further medical
treatment is futile.
For older children and adults, the physical examination must show
absence of cerebral and brain stem function, no response to deep central
pain, and absence of complicating conditions such as hypothermia or
hypotension. Findings consistent with absence of brain stem function are
dilated and nonreactive pupils, absent corneal reflexes, absent
oculocephalic (doll's eyes) reflex, absent oculovestibular reflex, and absent
oropharyngeal (gag) reflex.
In addition to these, the apnea test is used to assess the function of the
medulla. Brain death is confirmed if the patient has no spontaneous
respirations after allowing the PaCO2 to reach greater than 60
(hypercapnia of this degree will always produce spontaneous respirations
in a patient with a functioning brain stem). If a patient has extensor
(decerebrate) or flexor (decorticate) posturing in response to deep central
pain, then information from the brain stem is still being transmitted down
through the spinal cord which is incompatible with a diagnosis of brain
death.
Additionally, a patient should be free of any complicating condition that
may simulate brain death. Such conditions include hypothermia,
hypotension, intoxication, anoxia, immediate postresuscitation state, and
patients emerging from a pentobarbital coma. Certain observation periods
ranging from 6 to 24 h may also be warranted depending on the specific
circumstances.
For children less than 5 years of age coma and apnea must coexist, and
there must be absence of brainstem function on physical examination.
Additional criteria include two examinations and two negative
electroencephalograms (EEG) 48 h apart for children age 7 days to 2
months, two examinations and two negative EEGs 24 h apart for children
age 2 months to 12 months, and an interval of 12 h between examinations
and EEGs for children age 12 months to 5 years. Besides EEG, other
confirmatory tests for diagnosing brain death include cerebral angiography
and radionuclide blood flow studies. These studies may be helpful in
patients with severe congestive heart failure or chronic obstructive
pulmonary disease where the apnea test is invalid, in patients with severe
facial trauma which would preclude cranial nerve testing, in patients
coming out of a pentobarbital coma, and in allowing more expedient organ
donation.
Characteristics of fulminant hepatic failure are as
follows:
I. It is rarely a medical emergency.
II. It is a clinical syndrome representing a final
common pathway for a wide variety of diseases.
III. It is most commonly because of alcohol abuse.
IV. It has pathognomonic features making the
diagnosis evident.
V. Sometimes may not have an identifiable cause.
A. III, IV
B. III, V
C. II, III
D. IV, V
E. II, V
Answer E.
Explanation: Acute fulminant hepatic failure is an
uncommon manifestation of liver disease that constitutes a
medical emergency. It is because of loss of hepatic
parenchyma secondary to a given insult and carries a grave
prognosis.
Which of the following dietary changes will
improve the Child-Pugh classification in endstage liver patients with protein-calorie
malnutrition?
A. lactoalbumin supplementation
B. oral intake of fat soluble vitamin K
C. maltodextrin supplementation
D. branched chain amino acid
supplementation
E. long-term parenteral nutrition
Answer D.
Explanation: Malnutrition in the end-stage liver failure patient plays a
significant role in outcome. The nutritional deficiencies are mainly because
of protein malnutrition and have been shown to be an independent risk
factor for mortality and life-threatening complication. The factors causing
the protein loss and failure of intake are the hypermetabolism associated
with end-stage liver failure necessitating a greater protein intake. A protein
load in these patients can lead to worsening encephalopathic changes.
Branched chain amino acid supplements have been used successfully in
cirrhotic patients to increase the protein intake to combat nitrogen losses,
while avoiding encephalopathic changes.
Long-term studies assessing the benefits of oral branched chain amino
acid supplementation in these patients have found that there is no
statistically significant difference in mortality. When compared with
equivalent caloric and protein oral supplementation there have been
shown to be significant improvements in Child-Pugh score, number of
required hospital admissions, bilirubin, anorexia, and health-related quality
of life. Nutritional support with branched chain amino acids has improved
anthropometric measurements in this subset of malnourished patients and
this correction has been proven to prolong life.
The major downside to branched chain amino acid supplementation is that
the supplements are notoriously unpalatable and this had led to
noncompliance and patient withdrawal from trials.
A 25-year-old female is brought to the local emergency room
with signs and symptoms compatible with encephalopathy.
Her family members report that she had been jaundiced over
the past few days and had recently separated with her
boyfriend. She had been otherwise healthy and on no
medications. The most likely etiology and best treatment
course include the following.
I. Hepatic failure in this setting is usually secondary to viral
hepatitis.
II. The patient should receive broad-spectrum prophylactic
antibiotics
III. Steroid therapy should be started immediately.
IV. An evaluation for liver transplantation should be done as
soon as possible.
V. Acetylcysteine should be administered.
A. II, IV B. I, V C. I, II, III D. III, V E. IV, V
Answer E.
Explanation: The time course of fulminant hepatic failure has an etiologic and
prognostic significance. An illness of 1 week or less before the development of
failure is usually suggestive of hepatic ischemia or acetaminophen toxicity. On the
other hand, illness longer than 4 weeks is more likely the result of viral hepatitis
or hepatic failure of unknown etiology.
Patients who are ill for more than 8 weeks before they develop encephalopathy
have a higher chance of developing portal hypertension, whereas patients with
illness of shorter duration (<4 weeks) are more likely to develop cerebral edema.
Encephalopathy preceded by 1 week of jaundice is a poor prognostic indicator.
Because of its easy availability, acetaminophen is a commonly used drug for
suicide. This is also a problem with other over-the-counter remedies that contain
acetaminophen as an active ingredient.
Answer E.
Although infectious complication may develop in up to 80% of patients with
fulminant hepatic failure, prophylactic antibiotic therapy is still controversial. A
wide variety of therapies have been proposed and used for the treatment of this
disease, including corticosteroids, prostaglandins, and exchange transfusion, yet
none have proved efficacious. Only the development of liver transplantation has
allowed the salvage of patients with irreversible liver failure. Patients should
therefore be evaluated for liver transplantation as soon as possible and placed on
the transplant waiting list. Treatment is otherwise supportive and includes
prophylaxis for gastrointestinal bleeding in the setting of coagulopathy, correction
of hypoglycemia, intracranial pressure (ICP) monitoring of intracranial
hypertension, along with osmotherapy and barbiturates, hemodynamic
monitoring and organ-directed support if multiorgan failure develops.
Surveillance cultures should be routinely obtained and if infection is suspected,
empirical therapy should be tailored to local hospital antimicrobial sensitivities
and should cover Staphylococcus and gram-negative aerobes.
Treatment of a 3-mm displaced fracture
of the anterior wall of the frontal sinus
is
(A) Observation
(B) Antibiotics alone
(C) Open reduction
(D) Open reduction, demucosalization,
and packing of fat into the sinus
Answer C.
Explanation: The most common surgical approach to the frontal sinus is
through a coronal incision. Treatment of frontal sinus fractures is
predicated on the number of walls involved and the status of the
nasofrontal duct (Fig. 44-34). In nondisplaced anterior wall fractures, no
treatment is indicated. If the anterior wall is displaced, then elevation and
recontouring of the anterior table is executed. The patient should be
observed for any sinus opacification or obstruction. If the nasofrontal duct
is involved in the fracture, one can assume that this is a dysfunctional
sinus. Therefore, the sinus must be demucosalized, the nasofrontal duct
must be plugged with bone graft, and sinus cavity obliterated with
cancellous bone or fat. The technique of frontal sinus exenteration or
removal of the anterior table, with demucosalization plugging of the ducts,
is an antediluvian procedure not routinely performed because of the
significant contour deformity. (See Schwartz 8th ed., Chapter 44,
Maxillofacial Trauma.)
Which of the following is the most common
nerve deficit after resection of a poststyloid
compartment parapharyngeal
neurilemmoma?
(A) ptosis
(B) painful shoulder syndrome
(C) deviation of tongue to the operated side
(D) voice change or hoarseness
(E) corneal exposure
Answer A.
Explanation: The parapharyngeal space (PPS) can be thought of as an
inverted pyramid. The boundaries of this space are the base of skull
superiorly and the hyoid bone inferiorly. The space itself is deep to the
pharyngeal mucosa and superficial to the carotid sheath and it
communicates with the submandibular space. It can be divided into a
prestyloid and poststyloid or retrostyloid space by the syloid muscles and a
band of fascia from the tensor veli palatini. These spaces are important
when discussing tumor pathology and surgical approaches. The prestyloid
space contains fat, the mandibular branch of the facial nerve, the pterygoid
venous plexus, whereas the poststyloid space contains cranial nerves IX–
XII, the cervical sympathetic chain and the internal carotid artery and
internal jugular vein (IJV).
The differential of masses in the PPS is large but can be broken into four
categories: salivary gland tumors, neurogenic tumors, lymph node
enlargement, or miscellaneous tumors. Patients can present with
symptoms of airway obstruction from poststyloid masses, pain or cranial
nerve palsies of nerves in the PPS. Patients can also present with a
unilateral serous otitis media from Eustachian tube dysfunction. A CT scan
and/or MRI would be the initial test(s) of choice to delineate between pre-
All of these are part of the oral cavity
except
(A) floor of mouth
(B) soft palate
(C) base of tongue
(D) upper gingivae
(E) retromolar trigone
Answer C.
Explanation: The oral cavity is bounded by the vermilion border of the lips
and the junction of the hard and soft palate and circumvallate papillae. It
can be thought of having eight subunits: lips, buccal mucosa, floor of
mouth, anterior two-thirds of the tongue (i.e., oral tongue), upper and lower
alveolar ridges, hard palate, and retromolar trigone. The retromolar trigone
is a triangular spaced area from the distal surface of the last molar tooth to
the maxillary tuberosity. This area is important in cancer spread as the
mucosa of the mandible is tightly adherent to the underlying periosteum
and therefore a weak barrier to tumor extension. The vestibule is the area
lateral to the alveolar ridges and the oral cavity proper the area medial to
the teeth. The layers of the cheek itself from superficial to deep are as
follows: skin, subcutaneous tissue, the buccinator muscle, the buccinator
fat pad, the pharyngobuccal fascia, and the mucosa/lip complex. The
salivary ducts traverse the mucosa to drain into the oral cavity. These
include Stensen's duct of the parotid gland, the papilla of which is located
lateral to the second molars; Wharton's duct of the submandibular gland
which is found in the midline floor of mouth adjacent to the frenulum of the
tongue; and ducts of Rivinius of the sublingual gland which drain into the
floor of mouth or into Wharton's duct itself.
A 20-year-old man involved in an altercation presents to the ER
with epistaxis and nasal airway obstruction. When inspecting his
nose externally, you feel crepitus when moving the nasal bones
and mild flattening of the dorsum; there is no active bleeding. On
anterior rhinoscopy, you see an ecchymotic, swollen area on either
side of the caudal septum (Fig. 14-6). The next step in
management would be to
(A) reduce the nasal fracture externally and employ an external
nasal splint
(B) place internal nasal splints to stabilize the fracture
(C) drain the septal hematoma
(D) place anterior nasal packing to treat the epistaxis
(E) get facial x-rays if they were not already performed
Answer C.
Explanation: A history of trauma to the nose with epistaxis should raise
concern for a nasal fracture. Signs of crepitus of the nasal cartilaginous
and bony framework and obvious external deformity are virtually
pathognomic for a nasal fracture. The nasal bone is the most frequently
fractured facial bone. Diagnosis rests on the physical examination
especially after topical decongestion; x-rays have not been helpful in
adding to diagnostic accuracy. In nearly 50% of cases, nasal x-rays may
not reveal a fracture when one is actually present. Photographic
documentation is important, however. A careful rhinoscopic examination
should be performed as there are few injuries and/or complications
associated with nasal trauma to the nose that require immediate repair or
attention. One of these is the septal hematoma (Fig. 14-6).
A septal hematoma presents with nasal airway obstruction, usually
bilaterally. Less often do patients with a septal hematoma present with
epistaxis. The hematoma develops in the plane between the
perichondrium of the septal cartilage and the cartilage itself. As the
cartilage receives its blood supply from the perichondrium, the hematoma
causes ischemic injury and eventually degeneration of the cartilaginous
septum. A devastating cosmetic and functional consequence of this is the
A 60-year-old male with a history of
hypertension and coronary artery
disease presents to the ER with steady
bleeding from the right nare. He is on
aspirin 325 mg a day as well as Plavix;
he had a percutaneous transluminal
coronary angioplasty (PTCA) with
stenting 5 years ago. You first examine
him and are not sure where the
bleeding is arising from so you place
an anterior nasal pack. He has no
bleeding until 20 min later, but then
Answer C.
Explanation: Epistaxis or nosebleeding is one of the most common ear,
nose and throat (ENT) emergencies. The role of the nose in humidification,
filtration and warming of inspired air and its copious blood supply all put it
at risk for bleeding. Epistaxis more commonly occurs in older individuals
because of vessel wall aging with fibrosis and slower vasoconstriction and
in the winter months because of cold, dry air exposure. Other risk factors
include trauma (nose picking, most common in children), nasal sprays
including nasal steroids, intranasal or sinus tumors, allergies, medications
such as antiplatelet agents and anticoagulants, and anatomic deformities
such as septal deviation. Systemic factors and diseases putting patients at
epistaxis risk include hypertension, hereditary hemorrhagic telangiectasia
(Osler-Weber-Rendu disease, an autosomal dominant disease with
associated mucosal telangiectasias and pulmonary AVMs), von Willebrand
disease, hemophilia, nutritional deficiencies, alcohol abuse with associated
hepatic disease, and lymphoreticular disorders or malignancies.
Epistaxis most commonly occurs in the anterior portion of the nasal cavity,
specifically the septum and the area known as Kiesselbach's plexus (in
90%). This area is particularly susceptible to trauma and drying effects.
The first step in management of epistaxis is fluid resuscitation and control
Nasal blood supply. Major nasal blood
vessels and their relative positions are
depicted. Note that the nasal sept um has
been reflected superiorly. A, Anterior
ethmoidal artery; B, Posterior ethmoidal
artery; C, Posterior septal nasal artery; D,
Lateral nasal artery; E, Sphenopalatine
artery; F, Sphenopalatine foramen; G,
Greater palatine foramen; H, Greater
palatine artery; I, Incisive canal.
A 50-year-old man has a 2-day history of headaches
and of proptosis with failing vision in the right eye.
Vision has been reduced to light perception only and
the globe is displaced inferior and laterally. Rhinoscopy
shows swelling in the middle meatus with some
purulence. The next step in management would be to
(A) obtain a CT scan of the orbits and sinuses and
immediate ethmoidectomy
(B) IV aqueous penicillin G, 2 million U every 4 h
(C) IV levaquin 500 mg every 24 h
(D) immediate exploration of the orbits
(E) oral dexamethasone, 4 mg daily for 1 week
Answer A.
Explanation: The most common complication of acute sinusitis
necessitating immediate operative intervention involves the eye. All the
sinuses can be culprits of orbital complications but the ethmoid is the most
common because of its adjacency. The indication in this patient to operate
immediately would be the visual acuity change as complications can lead
to blindness. As the ethmoids are the culprit, decompressing the infection
or abscess if present can be performed via the lamina papyracea, the
medial wall of the orbit. Infections spread by direct extension and
thrombophlebitis of ethmoidal veins. Other complications may include
neurologic infections: subdural and epidural abscesses and meningitis.
Orbital complications are stratified by the Chandler classification system.
Stage I is simply inflammatory edema or preseptal cellulitis (orbital septum
of the eyelid) of the lids and extraocular muscles are not involved. Stage II
indicates orbital cellulitis with edema of the contents of the orbit. The first
two stages should be aggressively treated with medical therapy with
antibiotics against Streptoccocus pneumoniae and Haemophilus
influenzae to prevent progression to stage III. Stage III is the subperiosteal
abscess which is beneath the periosteum of the lamina papyracea; the
globe is displaced inferolaterally and vision is affected. Stage IV is an
Orbital complications are typically treated with an external approach rather
than and endoscopic approach although the trend is changing. Since the
ethmoid sinuses are the most frequently involved, at minimum, an external
ethmoidectomy is performed with removal of a portion of the lamina
papyracea.
Acute bacterial rhinosinusitis is diagnosed by the symptomatology of nasal
congestion and rhinorrhea lasting for 7–14 days. Other symptoms include
facial pain or dental pain, headache, fever and malaise. Anterior
rhinoscopy may reveal unilateral or bilateral purulent drainage and
tenderness on palpation of soft tissue over the sinuses. Various processes
may lead to acute or chronic sinusitis. The first is obstruction of sinus ostia
which can be caused by anatomic factors (septal deviation), edema from
allergens or polyps. The second process is ciliary dysfunction either
primary or acquired such as after a viral upper respiratiory infection (URI).
The last is changes in mucus quality or quantity systemic factors may
include steroid use, diabetes or immune compromise in general.
Nosocomial sinusitis may be caused by indwelling nasogastric catheters or
nasotracheal intubation. A critically ill patient may present with a fever of
unknown origin; acute rhinosinusitis should be given careful consideration,
usually with an original or reconstructed coronal CT scan (optimal images
for sinuses). These patients should be treated for gram-positive and gram-
An 80-year-old woman who is a nursing home patient is
brought to the hospital with a GI bleed. She ultimately
undergoes a left hemicolectomy for diverticulosis and
has a lengthy postoperative ileus. Postoperative day 5
she complains of a sour taste in her mouth and her right
cheek feeling warm and very tender. You notice a
swelling in the parotid area and that she is febrile at the
time. All of the following would be part of the treatment
of this disorder except:
(A) IV antibiotics against S. aureus
(B) heat application to the area over the parotid
(C) IV hydration
(D) lemon drops
(E) cannulation of the right Stensen's duct with drainage
Answer E.
xplanation: This case is typical of acute bacterial parotitis or sialadenitis.
This disorder occurs in individuals with dehydration from any cause. The
dehydration may be a result of being NPO for an extended period of time
or in the patient being in a relative state of anorexia after surgery and
patients who have undergone abdominal procedures are at the most risk.
Chronic and/or debilitating illnesses also may predispose a patient to
development of acute parotitis such as in the case of a nursing home
patient. Other causes may be radiation, chemotherapy or
immunosuppression in general, medications with anticholinergic side
effects, and Sjogren's disease. The incidence is reported at approximately
1 in 1–2000 operative procedures. Acute parotitis presents with symptoms
of pain, erythema, and diffuse enlargement of the gland which is usually
unilateral; gentle milking of Stensen's duct with manual pressure on the
gland and intraorally causes purulent exudate to be expressed from the
orifice.
The pathophysiology of acute parotitis is retrograde bacterial infection
through Stensen's duct. Mucoid saliva which has a high molecular weight
glycoprotein and sialic acid has superior bacteriostatic activity because of
the ability to trap bacteria. Mucoid saliva also has a higher lysozyme and
Which of the following statements
concerning cleft lip and cleft palate is
true?
(A) Cleft lip is a midline failure of lip
closure.
(B) Development of cleft lip is related to
environmental factors and not to familial
tendencies.
(C) Deformities of the nose occur in
approximately 50% of patients who have
a cleft lip.
(D) Middle ear infections are common
Answer D.
Explanation: The incidence of cleft palate, cleft lip, or both is variously
reported as 1 in 1000 to 1 in 2500 live births in the United States. If a
parent or sibling has a cleft lip, the chance of a subsequent child's being
born with the same defect is higher. Cleft lip may be unilateral or bilateral
when the nasomedial and nasolateral processes fail to unite during
embryologic development. Cleft palate is due to isolated failure of palatal
process fusion. Almost all cleft lips, even minor ones, are associated with
nasal distortions, and many plastic surgeons advocate rhinoplasty at the
time of repair of the cleft lip. Malocclusion is a uniform problem, and all
patients with cleft palates have drainage problems of the middle ear, which
may lead to recurrent ear infections. Normal speech is achieved in more
than 75% of cases of corrective surgery in which the cleft palate is closed
entirely by the age of 12 to 14 months. (See Schwartz 7th ed.)
What study is used to monitor
response to therapy for malignant
otitis externa?
(A) technetium-99 scan
(B) gallium-67 scan
(C) CT scan
(D) MRI
(E) culture
Answer B.
Explanation: Malignant otitis externa (or necrotizing otitis externa, NOE)
does not refer to a neoplastic process but rather a potentially lifethreatening infectious external ear infection. Though newer antibiotics have
somewhat decreased the incidence, there are certain patient groups at
risk. These at-risk groups are the elderly, diabetics, and the
immunocompromised. In fact, up to 80% of cases are found in diabetics.
NOE is associated with a skull base osteomyelitis which is the source of its
lethal nature.
Diagnosis rests on a few key symptoms and signs. These include a
persistent and severe otalgia that has lasted for more than 1 month,
purulent otorrhea with granulation tissue for several weeks,
immunocompromised because of age, diabetes or other condition and a
lower cranial neuropathy of nerves VII, IX, X, or XI. The granulation tissue
is seen at the posteroinferior aspect of the external auditory canal at the
tympanomastoid suture line. The granulation tissue seen in an AIDS
patient is less exuberant but suspicion should be maintained, as these
patients are usually severely immunocompromised. Granulation tissue can
be seen in severe otitis externa when there is a tympanic membrane
perforation. SCC can present in a similar fashion, so biopsies should be
What structure passes through the
foramen ovale?
(A) infraorbital nerve
(B) V3
(C) meningeal artery
(D) sphenopalatine artery
(E) V2
Answer B.
Explanation: The skull base is a complex anatomical region which houses
multiple vital structures. Any disease process in this region has the
potential to affect the function of the various contents of the skull base
foramina. The skull base can be simplified by separating it into an anterior,
middle, and posterior cranial fossa (Fig. 14-27).
FIG. 14-27
Skull base foramina from internal aspect. (Source: Reprinted from Lee, et
al. Essential Otolaryngology Head and Neck Surgery, 8th ed., copyright
2003, with permission from McGraw-Hill).
The anterior cranial fossa (ACF) is the most shallow. The main component
is the frontal bone and the floor is chiefly composed of the orbital plates of
the frontal bone and ends at the anterior border of the greater wing of
sphenoid. The fovea ethmoidalis is the portion of the ethmoid sinuses that
forms part of the ACF floor. The main foramina of the ACF are those in the
cribriform plate which transmit the olfactory nerve axons. Other important
foramina are those for the anterior and posterior ethmoidal neurovascular
bundles. The optic nerve is located 5–6 mm behind the posterior ethmoidal
artery. A vestigial foramen cecum is seen between the crista galli and
frontal crest and in 1% it is open and transmits a nasal emissary vein.
Answer B.
Explanation: The skull base is a complex anatomical region which houses
multiple vital structures. Any disease process in this region has the
potential to affect the function of the various contents of the skull base
foramina. The skull base can be simplified by separating it into an anterior,
middle, and posterior cranial fossa (Fig. 14-27).
FIG. 14-27
Skull base foramina from internal aspect. (Source: Reprinted from Lee, et
al. Essential Otolaryngology Head and Neck Surgery, 8th ed., copyright
2003, with permission from McGraw-Hill).
The anterior cranial fossa (ACF) is the most shallow. The main component
is the frontal bone and the floor is chiefly composed of the orbital plates of
the frontal bone and ends at the anterior border of the greater wing of
sphenoid. The fovea ethmoidalis is the portion of the ethmoid sinuses that
forms part of the ACF floor. The main foramina of the ACF are those in the
cribriform plate which transmit the olfactory nerve axons. Other important
foramina are those for the anterior and posterior ethmoidal neurovascular
bundles. The optic nerve is located 5–6 mm behind the posterior ethmoidal
artery. A vestigial foramen cecum is seen between the crista galli and
frontal crest and in 1% it is open and transmits a nasal emissary vein.
Pott's puffy tumor is seen most
commonly with which of these
conditions?
(A) otitis media
(B) frontal sinus fracture
(C) ethmoid sinusitis
(D) bacterial pharyngitis
(E) cervical spinal infection
Answer B.
Explanation: Pott's puffy tumor is a term used to describe a soft tissue
swelling because of a subperiosteal abscess over the region of the frontal
sinus. This occurs when the anterior table of the frontal sinus is involved in
an osteomyelitic process which is usually because of a sinusitis but may
be a result of a mucocele from a fracture. The offending organism in many
cases is S. aureus. A malignancy of the frontal sinus, though exceedingly
rare, should be considered in the differential. The pathophysiologic of
development of a mucocele with subsequent osteomyelitis is an obstructed
frontal sinus outflow tract. This occurs by thrombophlebitis of the diploic
veins of the frontal bone or via direct extension.
The treatment of frontal sinus fractures depends on which table(s) of bone
are fractured and the degree of displacement or comminution.
Nondisplaced noncomminuted anterior table fractures can usually be
observed unless CT scan shows persistent opacification, in which case
endoscopic exploration or trephination of the sinus may be warranted.
Displaced anterior table fractures produce obvious cosmetic deformity and
can be reduced with low profile miniplates and preservation of as much
native bone as possible. Nondisplaced posterior table fractures that do not
cause CSF leaks can be observed with antibiotic treatment. Indications for
Answer B.
Mucoceles can develop several years after the initial
fracture, the average being 7.5 years. Other
complications of frontal sinus fractures include sinusitis,
headache, forehead numbness (from supraorbital or
supratrochlear nerve trauma), meningitis, brain
abscess, cosmetic deformity, and diplopia and eye
pulsations in the event of a carotid-cavernous fistula.
Other intracranial complications of sinusitis in general
include meningitis, epidural, subdural and intracerebral
abscesses, and superior sagittal sinus thrombosis as
well as Pott's puffy tumor. Pott's abscess is an eponym
to describe tuberculosis of the spine and the other
answer choices are merely distractors.
A 47-year-old man is brought to the physician's office by his wife
who is having difficulty sleeping because of her husband's
extremely loud snoring. He does complain of headaches and
daytime sleepiness as well as some irritability. On examination, his
collar size is 18 in. and he is moderately obese. In addition you
note a septal deviation to the right and an elongated redundant
uvula and posterior pharyngeal mucosa. You obtain a
polysomnogram because you suspect sleep apnea: the patient's
RDI is 40 with a low saturation of 80%. Appropriate treatment
options include all of the following except:
(A) nasal continuous positive airway pressure (CPAP)
(B) septoplasty with uvulopalatopharyngoplasty (UPPP)
(C) encourage weight loss as the sole treatment
(D) orthodontic devices in conjunction with CPAP
(E) tracheostomy
Answer C.
Explanation: In the United States, OSA has a prevalence of 4% in men
and 2% in women. There are several systemic consequences to sleep
apnea including hypertension, myocardial infarction, and stroke. Patients
with sleep apnea have three to seven times the risk of having motor
vehicle accidents. As a result of these statistics, sleep apnea is being
diagnosed earlier and treated aggressively. There is a continuum of sleep
disordered breathing which ranges from sleep apnea to the Pickwickian's
syndrome. OSA is caused by an obstruction at any level of the upper
airway above the glottis. The muscle relaxation occurring in the deeper
stages of sleep occurs in the upper airway as well and patients
predisposed to OSA have excess tissue in the upper airway, causing an
airway collapse during inspiration. The patient is then awakened by
desaturation, signaled as a snorting or gasping noise, and then resumes
the pattern.
The RDI is the respiratory disturbance index which is obtained by
polysomnography. An RDI of greater than 5 is abnormal. Apnea itself is
defined as cessation of airflow for at least 10 s, and hypopneas are
desaturations without complete cessation. The RDI is the number of
apneas and hypopneas in 1 h. This measure allows stratification of
Le Fort II fracture entails injuries to all
of the following EXCEPT
(A) Medial wall of the orbit
(B) Alveolus
(C) Zygomaticomaxillary articulation
(D) Nasofrontal buttress
(E) Mandible
Answer A.
Explanation: Le Fort I fractures occur transversely across the alveolus,
above the level of the teeth apices. In a pure Le Fort I fracture, the palatal
vault is mobile while the nasal pyramid and orbital rims are stable. The Le
Fort II fracture extends through the nasofrontal buttress, medial wall of the
orbit, across the infraorbital rim, and through the zygomaticomaxillary
articulation. The nasal dorsum, palate, and medial part of the infraorbital
rim are mobile. The Le Fort III fracture is also known as craniofacial
disjunction. The frontozygomaticomaxillary, frontomaxillary, and frontonasal
suture lines are disrupted. The entire face is mobile from the cranium. It is
convenient to conceptualize complex midface fractures according to these
patterns; however, in reality, fractures reflect a combination of these three
types. (See Schwartz 8th ed., Chapter 17, Trauma of the Head and Neck.)
The overall success rate of cervical
myotomy for patients with a
pharyngoesophageal swallowing
dysfunction is
(A) 20%
(B) 35%
(C) 65%
(D) 90%
Answer C.
Explanation: The more liberal application of myotomy to problems of the
oropharyngeal phase of swallowing has resulted in an overall success rate
in the relief of symptoms of only 64%. When patients are selected using
radiographic or motility markers of disease as outlined above, it is unusual
for patients not to see benefit. (See Schwartz 8th ed., Chapter 24, Motility
Disorders of the Pharynx and Esophagus.)
A 53-year-old man has long-standing liver
cirrhosis secondary to hepatitis C infection.
The most appropriate screening regimen
should include
I. yearly CT scan of the abdomen
II. a liver biopsy
III. liver ultrasound
IV. AFP level
V. diagnostic laparoscopy
A. I, II B. III, V C. III, IV D. I, V E. II, III
Answer C.
Explanation: The risk factors for developing HCC are well documented
and include the presence of cirrhosis, chronic active viral hepatitis
associated with elevated AFP, age >50, male gender, family history of
HCC, and previously resected or ablated HCC. Once cirrhosis has
developed, HCC is estimated to occur at the rate of 1–4% per year. This
well-documented risk for developing HCC has led to the practice of
screening and surveillance of high-risk patients for HCC. Although there
are no randomized trials comparing surveillance with no surveillance, a
National Institute of Health Consensus Panel currently recommends the
use of ultrasonography and AFP levels for early detection of HCC in highrisk populations (Fig. 28-20).
FIG. 28-20Routine screening for HCC has been demonstrated not to be
cost-effective; however, the wide availability of color-flow or power Doppler
imaging provides a rapid noninvasive modality to serially examine the liver
in patients at risk for the development of HCC. The image depicted
illustrates an HCC lesion with a hallmark arterial feeding vessel visualized
by color-flow Doppler imaging.Serum markers other than AFP have no
proven efficacy for early detection of HCC, and ultrasound has a
reasonable sensitivity (60–78%) but is operator dependent. This
You are caring for a 50-year-old man who is 3
years out from a cadaveric renal transplant
for diabetic nephropathy. He had a single
episode of vascular rejection 1 month after
transplant, but otherwise has done well with
a baseline serum Cr of 1.8 mg/dL. He is
admitted to your service with increasing
fatigue, dyspnea with exertion, and acute
renal failure. His medications include
cyclosporine, mycophenolate mofetil,
prednisone, and nifedipine. On examination,
he is pale and weak with a BP of 150/95
Answer E.
Explanation: The patient has a hemolytic anemia (elevated LDH and
bilirubin), thrombocytopenia, and renal failure. Therefore, the patient meets
the criteria of HUS. The most important next step would be to obtain a
peripheral smear to document the presence of schiztocytes. HUS is
usually seen in children with bloody diarrhea associated with infection with
verotoxin producing E. coli from undercooked meat; however, HUS is also
seen in a variety of conditions including malignancy, scleroderma renal
crises, bone marrow transplantation, and the administration of the
immunosuppressants cyclosporine A and tacrolimus. In renal
transplantation, cyclosporine associated HUS may develop in up to 10% of
patients. Treatment is removal of the offending agent. Small case series
suggest that plasma exchange may be helpful.
Bibliography
Zarifian A, Meleg-Smith S, O'Donovan R, et al. Cyclosporine-associated
thrombotic microangiopathy in renal allografts. Kidney Int 1999;55:2457–
2466. [PubMed: 10354295]
A patient is brought to the ER 3 months
following liver transplantation. The patient
had been doing well until about 1 week prior
to admission when he because confused,
tremulous and complained of unsteadiness
and difficulty with vision. An MR scan showed
T2 hyperintense lesions in both occipital
lobes (Fig. 35-18). The most likely diagnosis is
FIG. 35-18
Axial T2-weighted MRI of the brain, revealing
white matter hyperintensity in both occipital
lobes.
Answer B.
Explanation: Cyclosporin toxicity can result in the posterior
leukoencephalopathy syndrome. There are a variety of acute illnesses that
can result in a reversible encephalopathy secondary to edema of the
cerebral white matter, most prominently in the occipital and posterior
parietal and temporal regions of the brain. Clinically the syndrome is
manifested by the subacute onset of headache, lethargy, confusion,
altered mental status, seizures, and difficultly with vision. The white matter
edema is visible as decreased attenuation on CT scans and hypointensity
on T1 and hyperintensity on T2 MR scans. Originally described in
encephalopathy associated with malignant hypertension and eclampsia of
pregnancy, the syndrome also occurs secondary to toxicity of cyclosporin
and other immunosuppressants. White matter edema results from
disruption of the blood-brain barrier. The mechanism for this disturbance is
not entirely clear in cases of immunosuppression. The syndrome can occur
with levels of drug in the therapeutic range and is probably the result of a
vasculopathy caused by the medication. The syndrome is reversible by
discontinuing or lowering the drug level. The radiologic abnormalities often
resolve completely within several weeks.
Creutzfeldt-Jakob's disease is a prion (proteinaceous infectious particle)
You are caring for a 50-year-old man who is 3
years out from a cadaveric renal transplant
for diabetic nephropathy. He had a single
episode of vascular rejection 1 month after
transplant, but otherwise has done well with
a baseline serum Cr of 1.8 mg/dL. He is
admitted to your service with increasing
fatigue, dyspnea with exertion, and acute
renal failure. His medications include
cyclosporine, mycophenolate mofetil,
prednisone, and nifedipine. On examination,
he is pale and weak with a BP of 150/95
A 33-year-old man tested positive for
antibody to HCV when donating blood. His
liver enzymes are within normal range. The
patient is otherwise healthy, feels well, and
denies any risk factors for HCV. His physical
examination is unremarkable and shows no
stigmata of chronic liver disease. Qualitative
polymerase chain reaction (PCR) testing is
positive for HCV RNA. Which of the following
actions is most appropriate for this patient?
A. reassure the patient that he does not have
chronic HCV infection and requires no further
Answer D.
Explanation: Screening recommendations for HCV are currently practiced
according to the Centers for Disease Control recommendations. The main
transmission mode is following initiation of injection drug use, whereas the
risk of sexual transmission is low. The accuracy of anti-HCV testing
(enzyme immunoassay) depends on the pretest probability of disease. The
predictive value in a patient with known parenteral exposure exceeds 90%,
whereas the predictive value of a positive anti-HCV test in blood donors
with a normal alanine aminotransferase (ALT) level and no risk factor for
HCV infection is less than 50%. In this situation, direct measurement of
HCV RNA by PCR assay is required.
Seventy to 85% of patients initially infected with HCV develop persistent
infection. In approximately 15% of patients, HCV infection resolves within
1–6 months, possibly because of HCV-specific T-cell function. Of those
patients with persistent infection, 20% have chronic viremia with normal
liver enzymes. In this group of patients, risk of progression to cirrhosis is
low. On the other hand, patients with a consistently or intermittently
elevated ALT level have a 20% risk of developing cirrhosis over 20 years.
The risk of hepatic decompensation manifesting as ascites, variceal bleed,
encephalopathy, or loss of hepatic synthetic ability averages about 3–5%
A 26-year-old surgery resident is evaluated for malaise,
fatigue, myalgia, and low-grade fever. He was previously well
and denies illicit drug use, transfusions, or recent travel. He
admits though to multiple needle stick accidents. Physical
examination reveals jaundice and hepatomegaly. Laboratory
studies are as follows: AST 1000 U/L, ALT 2000 U/L, INR 1.0,
total bilirubin 4 mg/dL, IgG anti-HAV positive, IgM anti-HAV
negative, HBsAg positive, IgG anti-HBc positive, IgM anti-HBc
positive, anti-HCV negative.
The most likely diagnosis is
A. acute hepatitis C
B. acute hepatitis B
C. chronic hepatitis A
D. chronic hepatitis B
E. chronic hepatitis C
Answer B.
Explanation: HBV is primarily transmitted by parenteral and mucous
membrane exposure to infectious body fluids such as blood, serum,
semen, and saliva. Risk factors include close personal or intimate
exposure to an infected household contact or sexual partner, intravenous
drug use, tattooing and body piercing, unapparent blood inoculations as
with shared razor blades, blood transfusion or exposure to blood products,
hemophilia and hemodialysis, and work in the health care profession.
Because of improved screening of blood donors, and educational efforts to
combat human immunodeficiency virus (HIV), the incidence of HBV
infection has declined in the United States since 1991.
Diagnosis of acute hepatitis depends on the results of specific antiviral
serology. Hospitalization is warranted for intractable symptoms of anorexia,
vomiting, or severe impairment of liver function. Other symptoms include
jaundice, weight loss, and malaise. Severe hepatic dysfunction manifests
as renal failure, metabolic acidosis, encephalopathy, variceal bleeding or
ascites.
In adults, more than 90% of HBV infection results in self-limited acute
hepatitis with subsequent resolution of the disease in 3–6 months.
Approximately 5% of patients will develop chronic hepatitis, and 1–2% will
progress to fulminant hepatitis.
IgM antibody to hepatitis B core antigen is the most specific marker for